Last visit was: 24 Apr 2024, 21:01 It is currently 24 Apr 2024, 21:01

Close
GMAT Club Daily Prep
Thank you for using the timer - this advanced tool can estimate your performance and suggest more practice questions. We have subscribed you to Daily Prep Questions via email.

Customized
for You

we will pick new questions that match your level based on your Timer History

Track
Your Progress

every week, we’ll send you an estimated GMAT score based on your performance

Practice
Pays

we will pick new questions that match your level based on your Timer History
Not interested in getting valuable practice questions and articles delivered to your email? No problem, unsubscribe here.
Close
Request Expert Reply
Confirm Cancel
SORT BY:
Date
Tags:
Show Tags
Hide Tags
User avatar
Manager
Manager
Joined: 20 Sep 2007
Posts: 58
Own Kudos [?]: 235 [72]
Given Kudos: 0
Send PM
Most Helpful Reply
GMAT Instructor
Joined: 04 Jul 2006
Posts: 960
Own Kudos [?]: 693 [28]
Given Kudos: 6
Location: Madrid
 Q51  V50
Send PM
General Discussion
User avatar
Manager
Manager
Joined: 29 Jan 2011
Posts: 160
Own Kudos [?]: 711 [0]
Given Kudos: 87
Send PM
Tutor
Joined: 16 Oct 2010
Posts: 14817
Own Kudos [?]: 64904 [4]
Given Kudos: 426
Location: Pune, India
Send PM
Re: Juan bought some books ,Gmat prep question [#permalink]
4
Kudos
Expert Reply
siddhans wrote:
kevincan wrote:
Juan bought some paperback books that cost $8 each and some hard cover books that cost $25 each. If Juan bought more than 10 paperback books , how many hardcover books did he buy.

1 The total cost of hardcover books that Juan bought was atleast 150$ He bought more than 6 hardcover books.
NOT SUFF

2 The total cost of all the books that Juan bought was less than $260
He spent at least $88 on paperback books, and thus less than 260 - 88 = $172 on hardcover books. This tells us he bought at most 6 hardover books

Together, we know he bought exactly 6 hardcover books. SUFF



I agree with 1

but in stmt 2 : How do we get 88 ? Juan bought atleast 10 paperback books so it has to 80 ...Next the total cost is less than 260 ..Lets say total cost is 100 ..

Then 100 - 80 = 20 ...So juan bought 0 hard cover books...

Combined do we get the answer still 6 here ???


As pointed out by esbee above, Juan bought more than 10 paperback books so he must have bought at least 11 paperbacks. This means he must have spent at least $88 on the paperbacks.

Statement 2 alone is not sufficient. All it tells us is that he spent less than $260. This means he spent less than $260 - $88 = $172 on hardcover books. He could have spent $88 + $50 = $138 OR $98 + $100 = $198 OR some other combination. We do not know how many hardcover books he bought but we do know that he could not have bought more than 6 since for 7 (or more) hardcover books, he would need $25*7 = $175 (or more). But he spent less than $172 on hardcover books. So he bought at most 6 hardcover books.

Using both statements together, statement 1 says that he bought at least 6 hardcover books and statement 2 says he bought at most 6 hardcover books. What does this mean? This means he must have bought exactly 6 hardcover books.
User avatar
Intern
Intern
Joined: 14 Jan 2012
Posts: 8
Own Kudos [?]: 7 [2]
Given Kudos: 13
GMAT 1: 600 Q45 V28
GMAT 2: 700 Q48 V38
Send PM
Re: Juan bought some paperback books that cost $8 each and some [#permalink]
2
Kudos
HI CAN SOMEONE PLEASE EXPLAIN WHY ADDING INEQUALITIES DOES NOT WORK IN THIS QUESTION

let the quantity of hardcover books be H and paperback P

Statement 1
25H is greater than or equal to 150
therefore H is greater than or equal to 6
Insufficient.

Statement 2

8(P)+25(H) < 260 AND
10< P

MULTIPLYING BY 8 AND ADDING IF WE SOLVE

H is less than 7.2 insuff

Statement 1 and 2 (H must be an integer)

and greater than or equal to 6,

therefore H can be 6 or 7.

E..I can't wrap around my head how this approach can be wrong? OA: C
Tutor
Joined: 16 Oct 2010
Posts: 14817
Own Kudos [?]: 64904 [2]
Given Kudos: 426
Location: Pune, India
Send PM
Re: Juan bought some paperback books that cost $8 each and some [#permalink]
2
Kudos
Expert Reply
GMAT98 wrote:
HI CAN SOMEONE PLEASE EXPLAIN WHY ADDING INEQUALITIES DOES NOT WORK IN THIS QUESTION

let the quantity of hardcover books be H and paperback P

Statement 1
25H is greater than or equal to 150
therefore H is greater than or equal to 6
Insufficient.

Statement 2

8(P)+25(H) < 260 AND
10< P

MULTIPLYING BY 8 AND ADDING IF WE SOLVE

H is less than 7.2 insuff

Statement 1 and 2 (H must be an integer)

and greater than or equal to 6,

therefore H can be 6 or 7.

E..I can't wrap around my head how this approach can be wrong? OA: C




Because the inequality 10 < P does not include all the information you have available. You know that P cannot be 10.5 i.e. a decimal. It must be an integer since it represents the number of paperbacks. So you might want to use 11<= P to get a tighter value for H.

8P + 25H < 260
88 <= 8P

When you add, you get 25H < 172 i.e. H < 6.9

All in all, algebra will always give you the correct result; but it may not be sufficient if you don't use reasoning as well. You got that H is less than 7.2 which is correct but it was not enough.
avatar
Intern
Intern
Joined: 18 May 2014
Posts: 42
Own Kudos [?]: 76 [3]
Given Kudos: 6
Location: United States
Concentration: General Management, Other
GMAT Date: 07-31-2014
GPA: 3.99
WE:Analyst (Consulting)
Send PM
Re: Juan bought some paperback books that cost $8 each and some [#permalink]
2
Kudos
1
Bookmarks
Statement 1: Juan spent at least $150 on Hardcovers. So that means he bought at least 6 harcovers. We have no upside limit though.

Insufficient.

Statement 2: Juan spent less than $260 total on books. This tells us at most that Juan spent $259 - $88 (minimum amount possible spent on paperbacks) = $171 maximum on hardcovers. 171 divided by 25 is 6 and change. So round down. The maximum number of harcovers that could be purchased is 6. This statement could also be true with any number of hardcovers below six being purchased. For example, 0 hardcovers + 12 paperbacks = $96, which is less than $260.

Insufficient.

Together:

Looking at Statement 1, we know that at least 6 hardcovers had to be purchased. Looking at Statement 2, we know that no more than 6 hardcovers could be purchased. So Juan must have purchased 6 hardcovers.
User avatar
Manager
Manager
Joined: 15 Aug 2013
Posts: 180
Own Kudos [?]: 331 [0]
Given Kudos: 23
Send PM
Re: Juan bought some paperback books that cost $8 each and some [#permalink]
VeritasPrepKarishma wrote:
siddhans wrote:
kevincan wrote:
Juan bought some paperback books that cost $8 each and some hard cover books that cost $25 each. If Juan bought more than 10 paperback books , how many hardcover books did he buy.

1 The total cost of hardcover books that Juan bought was atleast 150$ He bought more than 6 hardcover books.
NOT SUFF

2 The total cost of all the books that Juan bought was less than $260
He spent at least $88 on paperback books, and thus less than 260 - 88 = $172 on hardcover books. This tells us he bought at most 6 hardover books

Together, we know he bought exactly 6 hardcover books. SUFF



I agree with 1

but in stmt 2 : How do we get 88 ? Juan bought atleast 10 paperback books so it has to 80 ...Next the total cost is less than 260 ..Lets say total cost is 100 ..

Then 100 - 80 = 20 ...So juan bought 0 hard cover books...

Combined do we get the answer still 6 here ???


As pointed out by esbee above, Juan bought more than 10 paperback books so he must have bought at least 11 paperbacks. This means he must have spent at least $88 on the paperbacks.

Statement 2 alone is not sufficient. All it tells us is that he spent less than $260. This means he spent less than $260 - $88 = $172 on hardcover books. He could have spent $88 + $50 = $138 OR $98 + $100 = $198 OR some other combination. We do not know how many hardcover books he bought but we do know that he could not have bought more than 6 since for 7 (or more) hardcover books, he would need $25*7 = $175 (or more). But he spent less than $172 on hardcover books. So he bought at most 6 hardcover books.

Using both statements together, statement 1 says that he bought at least 6 hardcover books and statement 2 says he bought at most 6 hardcover books. What does this mean? This means he must have bought exactly 6 hardcover books.


Hi Karishma,

These exact type of problems give me a lot of trouble but I'm definitely getting better at them. I just get overwhelmed with information despite making charts etc and before I know it, it's 3+ minutes in and i'm stuck! What a complete waste of time.

Do you have a post that tackles on how to deal with these problems, perhaps algebraically.

Looking at the solution posted above, I can see how you came to the actual answer. That's never my problem. I can always FOLLOW the work, but for some reason, I can't start it myself. How do I fix this?

For this problem, I created a table but it ended up in shambles. Would really appreciate your help here.

Statement 1/2 insufficient solo. I'm always skeptical of C so I guessed E at the end of 3 minutes, like I said, horrible waste of time. My equations were as such.

Statement 1: (x=# of hc books)(25) < 150
Statement 2: (x)(25) + (11)(8)<260

I tried to see what value will make statement 2 valid. I noticed that 8(whatever number) has to end in a 0 to make this valid and it has to be greater than 88, so I had 120 and 160.

That made x only valid if the other value was 160, therefore, x had to 4. This is completely off than the explanation above. Where did I go wrong?
Tutor
Joined: 16 Oct 2010
Posts: 14817
Own Kudos [?]: 64904 [0]
Given Kudos: 426
Location: Pune, India
Send PM
Re: Juan bought some paperback books that cost $8 each and some [#permalink]
Expert Reply
russ9 wrote:
Hi Karishma,

These exact type of problems give me a lot of trouble but I'm definitely getting better at them. I just get overwhelmed with information despite making charts etc and before I know it, it's 3+ minutes in and i'm stuck! What a complete waste of time.

Do you have a post that tackles on how to deal with these problems, perhaps algebraically.

Looking at the solution posted above, I can see how you came to the actual answer. That's never my problem. I can always FOLLOW the work, but for some reason, I can't start it myself. How do I fix this?

For this problem, I created a table but it ended up in shambles. Would really appreciate your help here.

Statement 1/2 insufficient solo. I'm always skeptical of C so I guessed E at the end of 3 minutes, like I said, horrible waste of time. My equations were as such.

Statement 1: (x=# of hc books)(25) < 150
Statement 2: (x)(25) + (11)(8)<260

I tried to see what value will make statement 2 valid. I noticed that 8(whatever number) has to end in a 0 to make this valid and it has to be greater than 88, so I had 120 and 160.

That made x only valid if the other value was 160, therefore, x had to 4. This is completely off than the explanation above. Where did I go wrong?


I have discussed this here: https://www.gmatclub.com/forum/veritas-prep-resource-links-no-longer-available-399979.html#/2013/09 ... ct-answer/
Current Student
Joined: 30 Dec 2013
Status:preparing
Posts: 28
Own Kudos [?]: 26 [1]
Given Kudos: 26
Location: United Arab Emirates
Concentration: Technology, Entrepreneurship
GMAT 1: 660 Q45 V35
GMAT 2: 640 Q49 V28
GPA: 2.84
WE:General Management (Consumer Products)
Send PM
Juan bought some paperback books that cost $8 each and some [#permalink]
1
Kudos
neeraj.kaushal wrote:
Juan bought some paperback books that cost $8 each and some hardcover books that cost $25 each. If Juan bought more than 10 paperback books, how many hardcover books did he buy?

(1) The total cost of hardcover books that Juan bought was at least $150
(2) The total cost of all books that Juan bought was less than $260


1) 25h>=150
Sol : h>=150/25 => h>=6
it can be from 6 to +infinite. not a definite answer. so, NOT SUFFICIENT ALONE.

2) 8 p + 25 h < 260
sol: its been mentioned that p>10.

lets say that she bought only one hardcover, h=1, makes p=29

but "p" can be any number from 11 to 29 (price of p=8*11=88 to 8*29=232) : (lets say that she bought only one hardcover, h=1, makes p=29)
depending upon "p", value of "h" can vary from 1->6 (since p>10, so if p=11, h=6)
so this again is NOT SUFFICIENT ALONE.

NOW LETS TAKE BOTH statements 1 and 2:

1) h>=6
2)1<= h <=6

so h=6." C " is the answer
Director
Director
Joined: 26 Oct 2016
Posts: 510
Own Kudos [?]: 3378 [0]
Given Kudos: 877
Location: United States
Concentration: Marketing, International Business
Schools: HBS '19
GMAT 1: 770 Q51 V44
GPA: 4
WE:Education (Education)
Send PM
Re: Juan bought some paperback books that cost $8 each and some [#permalink]
From Stem : Price Paperback = $ 8
Price Hardcover = $ 25 , Number paperback >10 , P = { 11, 12, ....}
P and H are integers

St1>
25* Number Hardcover >=150
Number Hardcover > =6 Then , H = { 6, 7, ....}

Since question requires a unique value, INSUFFICIENT

St2> 8P + 25H < 260
Since Greatest Common Factor of { 8, 25, 260} is different from 1,
More than one solution , INSUFFICIENT

From St1 and St2 >

While P and H can only be integers

Plug the minimmum values for H and P
at 8P + 25H < 260,

Eg. ( P, H ) at 8P + 25H < 260,
( 11, 6 ) , 238 < 260, VALID
( 11, 7 ) 263 < 260, WRONG
( 12, 7 ) 271 < 260, WRONG

From this point, all other combinations will NOT comply the inequality.
Then , there is a unique value for H = 6

answer = C


To find the GCF of two numbers:
1) List the prime factors of each number.
2) Multiply those factors both numbers have in common. If there are no common prime factors, the GCF is 1.
Target Test Prep Representative
Joined: 14 Oct 2015
Status:Founder & CEO
Affiliations: Target Test Prep
Posts: 18756
Own Kudos [?]: 22049 [1]
Given Kudos: 283
Location: United States (CA)
Send PM
Re: Juan bought some paperback books that cost $8 each and some [#permalink]
1
Kudos
Expert Reply
neeraj.kaushal wrote:
Juan bought some paperback books that cost $8 each and some hardcover books that cost $25 each. If Juan bought more than 10 paperback books, how many hardcover books did he buy?

(1) The total cost of hardcover books that Juan bought was at least $150
(2) The total cost of all books that Juan bought was less than $260


We are given that Juan bought some paperback books that cost $8 each and some hardcover books that cost $25 each, and that he bought more than 10 paperback books. If we let p = the number of paperback books Juan bought and h = the number of hardcover books, the total cost of the paperback books is 8p, the total cost of the hardcover books is 25h, and the total cost of all books is 8p + 25h.

We need to determine the value of h, the number of hardcover books purchased by Juan.

Statement One Alone:
The total cost of the hardcover books that Juan bought was at least $150.

Using the information in statement one, we can create the following inequality:

25h ≥ 150

h ≥ 6

Thus, at least 6 hardcover books were purchased. However, we cannot determine the exact number of hardcover books purchased by Juan. Statement one alone is not sufficient to answer the question. We can eliminate answer choices A and D.

Statement Two Alone:

The total cost of all the books that Juan bought was less than $260.

Using the information in statement two, we can create the following inequality:

8p + 25h ≤ 260

We still do not have enough information to determine how many hardcover books were purchased by Juan.

Statements One and Two Together:

From the given information as well as our two statements, we know that p > 10, h ≥ 6, and that 8p + 25h ≤ 260.

Since p > 10, Juan, at minimum, purchased 11 paperback books. If we use 11 for p in the inequality 8p + 25h ≤ 260, then we have:

88 + 25h ≤ 260

25h ≤ 172

h ≤ 6 22/25

Recall that h ≥ 6, and since h has to be a whole number, Juan must have purchased 6 hardcover books.

Answer: C
Senior Manager
Senior Manager
Joined: 28 Jan 2017
Posts: 365
Own Kudos [?]: 78 [0]
Given Kudos: 832
Send PM
Juan bought some paperback books that cost $8 each and some [#permalink]
neeraj.kaushal wrote:
Juan bought some paperback books that cost $8 each and some hardcover books that cost $25 each. If Juan bought more than 10 paperback books, how many hardcover books did he buy?

(1) The total cost of hardcover books that Juan bought was at least $150
(2) The total cost of all books that Juan bought was less than $260

Dear IanStewart,

Do you recommend using algebraic approach in solving inequality for whole number items?

Let's #paperback books = p, and #hardcover books = h.
p > 10
h = ?

Statement 1: 25h ≥ 150 ---> h ≥ 6
Statement 2:

260 > 8p + 25h

Since
p > 10 *** Here's the trap I've fallen. It should be p ≥ 11***
8p > 80
260 > 8p + 25h > 80 + 25h
260 > 80 + 25h
7.2 > h

When combining both statements, I've got h = 6 or 7. (Hence, E -- most popular choice, btw)
The trap is very subtle.
GMAT Tutor
Joined: 24 Jun 2008
Posts: 4128
Own Kudos [?]: 9242 [1]
Given Kudos: 91
 Q51  V47
Send PM
Re: Juan bought some paperback books that cost $8 each and some [#permalink]
1
Kudos
Expert Reply
varotkorn wrote:
Do you recommend using algebraic approach in solving inequality for whole number items?


It very much depends on the question. Sometimes algebra is truly necessary, but any time a conceptual solution is available, it will be faster than an algebraic one.

In this question, there's no reason at all to spend time doing algebra. Each Statement is clearly insufficient alone, and using both, Juan spent at least $88 on paperbacks, and bought at least 6 hardcovers for $150. That already accounts for $238, and adding one more $25 hardcover would put him over $260, so he must have bought 6 hardcovers.
Senior Manager
Senior Manager
Joined: 14 Jul 2019
Status:Student
Posts: 478
Own Kudos [?]: 369 [0]
Given Kudos: 52
Location: United States
Concentration: Accounting, Finance
GMAT 1: 650 Q45 V35
GPA: 3.9
WE:Education (Accounting)
Send PM
Re: Juan bought some paperback books that cost $8 each and some [#permalink]
neeraj.kaushal wrote:
Juan bought some paperback books that cost $8 each and some hardcover books that cost $25 each. If Juan bought more than 10 paperback books, how many hardcover books did he buy?

(1) The total cost of hardcover books that Juan bought was at least $150
(2) The total cost of all books that Juan bought was less than $260


Info from stem : Juan spent at least $88 on paperback books.

1) Juan bought at least 7 hardcover books. That means, she spent at least $175 on hardcover books. Not sufficient

2) Juan spent at most 266 - 88 = $178 on hardcover books. She can bought any number of H.B from 1 to 7. Not sufficient.

Together, Juan bought 7 hardcover books. Sufficient.

C is the answer.
Intern
Intern
Joined: 02 Aug 2022
Posts: 41
Own Kudos [?]: 2 [0]
Given Kudos: 23
Send PM
Re: Juan bought some paperback books that cost $8 each and some [#permalink]
KarishmaB wrote:
siddhans wrote:
kevincan wrote:
Juan bought some paperback books that cost $8 each and some hard cover books that cost $25 each. If Juan bought more than 10 paperback books , how many hardcover books did he buy.

1 The total cost of hardcover books that Juan bought was atleast 150$ He bought more than 6 hardcover books.
NOT SUFF

2 The total cost of all the books that Juan bought was less than $260
He spent at least $88 on paperback books, and thus less than 260 - 88 = $172 on hardcover books. This tells us he bought at most 6 hardover books

Together, we know he bought exactly 6 hardcover books. SUFF



I agree with 1

but in stmt 2 : How do we get 88 ? Juan bought atleast 10 paperback books so it has to 80 ...Next the total cost is less than 260 ..Lets say total cost is 100 ..

Then 100 - 80 = 20 ...So juan bought 0 hard cover books...

Combined do we get the answer still 6 here ???


As pointed out by esbee above, Juan bought more than 10 paperback books so he must have bought at least 11 paperbacks. This means he must have spent at least $88 on the paperbacks.

Statement 2 alone is not sufficient. All it tells us is that he spent less than $260. This means he spent less than $260 - $88 = $172 on hardcover books. He could have spent $88 + $50 = $138 OR $98 + $100 = $198 OR some other combination. We do not know how many hardcover books he bought but we do know that he could not have bought more than 6 since for 7 (or more) hardcover books, he would need $25*7 = $175 (or more). But he spent less than $172 on hardcover books. So he bought at most 6 hardcover books.

Using both statements together, statement 1 says that he bought at least 6 hardcover books and statement 2 says he bought at most 6 hardcover books. What does this mean? This means he must have bought exactly 6 hardcover books.


Hi Karishma please correct me on my logic from statement one we get that number of hard books bought is > or = 6

Now statement two says 8p+25h<260
Now least number of paperbacks Juan could have bought is 11 this implies least amount of money he could have spent on paperback is 88 this maximum he could have spent on hardback would be less than 260-78= 172

25h<172 thus h< 172/25 this h< 6.8 as number of books bought has to be integer maximum number of books bought has less than or equal to 6

Together 1 and 2

Statement 1 hard books h =or> 6
Statement 2 less then or equal to 6

Thus 6

Posted from my mobile device
User avatar
Non-Human User
Joined: 09 Sep 2013
Posts: 32658
Own Kudos [?]: 821 [0]
Given Kudos: 0
Send PM
Re: Juan bought some paperback books that cost $8 each and some [#permalink]
Hello from the GMAT Club BumpBot!

Thanks to another GMAT Club member, I have just discovered this valuable topic, yet it had no discussion for over a year. I am now bumping it up - doing my job. I think you may find it valuable (esp those replies with Kudos).

Want to see all other topics I dig out? Follow me (click follow button on profile). You will receive a summary of all topics I bump in your profile area as well as via email.
GMAT Club Bot
Re: Juan bought some paperback books that cost $8 each and some [#permalink]
Moderator:
Math Expert
92900 posts

Powered by phpBB © phpBB Group | Emoji artwork provided by EmojiOne